Stay ahead of learning milestones! Enroll in a class over the summer!

G
Topic
First Poster
Last Poster
k a May Highlights and 2025 AoPS Online Class Information
jlacosta   0
May 1, 2025
May is an exciting month! National MATHCOUNTS is the second week of May in Washington D.C. and our Founder, Richard Rusczyk will be presenting a seminar, Preparing Strong Math Students for College and Careers, on May 11th.

Are you interested in working towards MATHCOUNTS and don’t know where to start? We have you covered! If you have taken Prealgebra, then you are ready for MATHCOUNTS/AMC 8 Basics. Already aiming for State or National MATHCOUNTS and harder AMC 8 problems? Then our MATHCOUNTS/AMC 8 Advanced course is for you.

Summer camps are starting next month at the Virtual Campus in math and language arts that are 2 - to 4 - weeks in duration. Spaces are still available - don’t miss your chance to have an enriching summer experience. There are middle and high school competition math camps as well as Math Beasts camps that review key topics coupled with fun explorations covering areas such as graph theory (Math Beasts Camp 6), cryptography (Math Beasts Camp 7-8), and topology (Math Beasts Camp 8-9)!

Be sure to mark your calendars for the following upcoming events:
[list][*]May 9th, 4:30pm PT/7:30pm ET, Casework 2: Overwhelming Evidence — A Text Adventure, a game where participants will work together to navigate the map, solve puzzles, and win! All are welcome.
[*]May 19th, 4:30pm PT/7:30pm ET, What's Next After Beast Academy?, designed for students finishing Beast Academy and ready for Prealgebra 1.
[*]May 20th, 4:00pm PT/7:00pm ET, Mathcamp 2025 Qualifying Quiz Part 1 Math Jam, Problems 1 to 4, join the Canada/USA Mathcamp staff for this exciting Math Jam, where they discuss solutions to Problems 1 to 4 of the 2025 Mathcamp Qualifying Quiz!
[*]May 21st, 4:00pm PT/7:00pm ET, Mathcamp 2025 Qualifying Quiz Part 2 Math Jam, Problems 5 and 6, Canada/USA Mathcamp staff will discuss solutions to Problems 5 and 6 of the 2025 Mathcamp Qualifying Quiz![/list]
Our full course list for upcoming classes is below:
All classes run 7:30pm-8:45pm ET/4:30pm - 5:45pm PT unless otherwise noted.

Introductory: Grades 5-10

Prealgebra 1 Self-Paced

Prealgebra 1
Tuesday, May 13 - Aug 26
Thursday, May 29 - Sep 11
Sunday, Jun 15 - Oct 12
Monday, Jun 30 - Oct 20
Wednesday, Jul 16 - Oct 29

Prealgebra 2 Self-Paced

Prealgebra 2
Wednesday, May 7 - Aug 20
Monday, Jun 2 - Sep 22
Sunday, Jun 29 - Oct 26
Friday, Jul 25 - Nov 21

Introduction to Algebra A Self-Paced

Introduction to Algebra A
Sunday, May 11 - Sep 14 (1:00 - 2:30 pm ET/10:00 - 11:30 am PT)
Wednesday, May 14 - Aug 27
Friday, May 30 - Sep 26
Monday, Jun 2 - Sep 22
Sunday, Jun 15 - Oct 12
Thursday, Jun 26 - Oct 9
Tuesday, Jul 15 - Oct 28

Introduction to Counting & Probability Self-Paced

Introduction to Counting & Probability
Thursday, May 15 - Jul 31
Sunday, Jun 1 - Aug 24
Thursday, Jun 12 - Aug 28
Wednesday, Jul 9 - Sep 24
Sunday, Jul 27 - Oct 19

Introduction to Number Theory
Friday, May 9 - Aug 1
Wednesday, May 21 - Aug 6
Monday, Jun 9 - Aug 25
Sunday, Jun 15 - Sep 14
Tuesday, Jul 15 - Sep 30

Introduction to Algebra B Self-Paced

Introduction to Algebra B
Tuesday, May 6 - Aug 19
Wednesday, Jun 4 - Sep 17
Sunday, Jun 22 - Oct 19
Friday, Jul 18 - Nov 14

Introduction to Geometry
Sunday, May 11 - Nov 9
Tuesday, May 20 - Oct 28
Monday, Jun 16 - Dec 8
Friday, Jun 20 - Jan 9
Sunday, Jun 29 - Jan 11
Monday, Jul 14 - Jan 19

Paradoxes and Infinity
Mon, Tue, Wed, & Thurs, Jul 14 - Jul 16 (meets every day of the week!)

Intermediate: Grades 8-12

Intermediate Algebra
Sunday, Jun 1 - Nov 23
Tuesday, Jun 10 - Nov 18
Wednesday, Jun 25 - Dec 10
Sunday, Jul 13 - Jan 18
Thursday, Jul 24 - Jan 22

Intermediate Counting & Probability
Wednesday, May 21 - Sep 17
Sunday, Jun 22 - Nov 2

Intermediate Number Theory
Sunday, Jun 1 - Aug 24
Wednesday, Jun 18 - Sep 3

Precalculus
Friday, May 16 - Oct 24
Sunday, Jun 1 - Nov 9
Monday, Jun 30 - Dec 8

Advanced: Grades 9-12

Olympiad Geometry
Tuesday, Jun 10 - Aug 26

Calculus
Tuesday, May 27 - Nov 11
Wednesday, Jun 25 - Dec 17

Group Theory
Thursday, Jun 12 - Sep 11

Contest Preparation: Grades 6-12

MATHCOUNTS/AMC 8 Basics
Friday, May 23 - Aug 15
Monday, Jun 2 - Aug 18
Thursday, Jun 12 - Aug 28
Sunday, Jun 22 - Sep 21
Tues & Thurs, Jul 8 - Aug 14 (meets twice a week!)

MATHCOUNTS/AMC 8 Advanced
Sunday, May 11 - Aug 10
Tuesday, May 27 - Aug 12
Wednesday, Jun 11 - Aug 27
Sunday, Jun 22 - Sep 21
Tues & Thurs, Jul 8 - Aug 14 (meets twice a week!)

AMC 10 Problem Series
Friday, May 9 - Aug 1
Sunday, Jun 1 - Aug 24
Thursday, Jun 12 - Aug 28
Tuesday, Jun 17 - Sep 2
Sunday, Jun 22 - Sep 21 (1:00 - 2:30 pm ET/10:00 - 11:30 am PT)
Monday, Jun 23 - Sep 15
Tues & Thurs, Jul 8 - Aug 14 (meets twice a week!)

AMC 10 Final Fives
Sunday, May 11 - Jun 8
Tuesday, May 27 - Jun 17
Monday, Jun 30 - Jul 21

AMC 12 Problem Series
Tuesday, May 27 - Aug 12
Thursday, Jun 12 - Aug 28
Sunday, Jun 22 - Sep 21
Wednesday, Aug 6 - Oct 22

AMC 12 Final Fives
Sunday, May 18 - Jun 15

AIME Problem Series A
Thursday, May 22 - Jul 31

AIME Problem Series B
Sunday, Jun 22 - Sep 21

F=ma Problem Series
Wednesday, Jun 11 - Aug 27

WOOT Programs
Visit the pages linked for full schedule details for each of these programs!


MathWOOT Level 1
MathWOOT Level 2
ChemWOOT
CodeWOOT
PhysicsWOOT

Programming

Introduction to Programming with Python
Thursday, May 22 - Aug 7
Sunday, Jun 15 - Sep 14 (1:00 - 2:30 pm ET/10:00 - 11:30 am PT)
Tuesday, Jun 17 - Sep 2
Monday, Jun 30 - Sep 22

Intermediate Programming with Python
Sunday, Jun 1 - Aug 24
Monday, Jun 30 - Sep 22

USACO Bronze Problem Series
Tuesday, May 13 - Jul 29
Sunday, Jun 22 - Sep 1

Physics

Introduction to Physics
Wednesday, May 21 - Aug 6
Sunday, Jun 15 - Sep 14
Monday, Jun 23 - Sep 15

Physics 1: Mechanics
Thursday, May 22 - Oct 30
Monday, Jun 23 - Dec 15

Relativity
Mon, Tue, Wed & Thurs, Jun 23 - Jun 26 (meets every day of the week!)
0 replies
jlacosta
May 1, 2025
0 replies
Brilliant guessing game on triples
Assassino9931   2
N a minute ago by Mirjalol
Source: Al-Khwarizmi Junior International Olympiad 2025 P8
There are $100$ cards on a table, flipped face down. Madina knows that on each card a single number is written and that the numbers are different integers from $1$ to $100$. In a move, Madina is allowed to choose any $3$ cards, and she is told a number that is written on one of the chosen cards, but not which specific card it is on. After several moves, Madina must determine the written numbers on as many cards as possible. What is the maximum number of cards Madina can ensure to determine?

Shubin Yakov, Russia
2 replies
Assassino9931
Saturday at 9:46 AM
Mirjalol
a minute ago
ISI UGB 2025 P5
SomeonecoolLovesMaths   4
N 26 minutes ago by Shiny_zubat
Source: ISI UGB 2025 P5
Let $a,b,c$ be nonzero real numbers such that $a+b+c \neq 0$. Assume that $$\frac{1}{a} + \frac{1}{b} + \frac{1}{c} = \frac{1}{a+b+c}$$Show that for any odd integer $k$, $$\frac{1}{a^k} + \frac{1}{b^k} + \frac{1}{c^k} = \frac{1}{a^k+b^k+c^k}.$$
4 replies
SomeonecoolLovesMaths
Yesterday at 11:15 AM
Shiny_zubat
26 minutes ago
ISI UGB 2025 P2
SomeonecoolLovesMaths   6
N 27 minutes ago by quasar_lord
Source: ISI UGB 2025 P2
If the interior angles of a triangle $ABC$ satisfy the equality, $$\sin ^2 A + \sin ^2 B + \sin^2  C = 2 \left( \cos ^2 A + \cos ^2 B + \cos ^2 C \right),$$prove that the triangle must have a right angle.
6 replies
SomeonecoolLovesMaths
Yesterday at 11:16 AM
quasar_lord
27 minutes ago
ISI UGB 2025 P6
SomeonecoolLovesMaths   3
N 29 minutes ago by Shiny_zubat
Source: ISI UGB 2025 P6
Let $\mathbb{N}$ denote the set of natural numbers, and let $\left( a_i, b_i \right)$, $1 \leq i \leq 9$, be nine distinct tuples in $\mathbb{N} \times \mathbb{N}$. Show that there are three distinct elements in the set $\{ 2^{a_i} 3^{b_i} \colon 1 \leq i \leq 9 \}$ whose product is a perfect cube.
3 replies
SomeonecoolLovesMaths
Yesterday at 11:18 AM
Shiny_zubat
29 minutes ago
Square number
linkxink0603   5
N 5 hours ago by linkxink0603
Find m is positive interger such that m^4+3^m is square number
5 replies
linkxink0603
May 9, 2025
linkxink0603
5 hours ago
Functions
Entrepreneur   5
N Today at 12:33 AM by RandomMathGuy500
Let $f(x)$ be a polynomial with integer coefficients such that $f(0)=2020$ and $f(a)=2021$ for some integer $a$. Prove that there exists no integer $b$ such that $f(b) = 2022$.
5 replies
Entrepreneur
Aug 18, 2023
RandomMathGuy500
Today at 12:33 AM
Logarithmic function
jonny   2
N Yesterday at 11:09 PM by KSH31415
If $\log_{6}(15) = a$ and $\log_{12}(18)=b,$ Then $\log_{25}(24)$ in terms of $a$ and $b$
2 replies
jonny
Jul 15, 2016
KSH31415
Yesterday at 11:09 PM
book/resource recommendations
walterboro   0
Yesterday at 8:57 PM
hi guys, does anyone have book recs (or other resources) for like aime+ level alg, nt, geo, comb? i want to learn a lot of theory in depth
also does anyone know how otis or woot is like from experience?
0 replies
walterboro
Yesterday at 8:57 PM
0 replies
Engineers Induction FTW
RP3.1415   11
N Yesterday at 6:53 PM by Markas
Define a sequence as $a_1=x$ for some real number $x$ and \[ a_n=na_{n-1}+(n-1)(n!(n-1)!-1) \]for integers $n \geq 2$. Given that $a_{2021} =(2021!+1)^2 +2020!$, and given that $x=\dfrac{p}{q}$, where $p$ and $q$ are positive integers whose greatest common divisor is $1$, compute $p+q.$
11 replies
RP3.1415
Apr 26, 2021
Markas
Yesterday at 6:53 PM
Incircle concurrency
niwobin   0
Yesterday at 4:28 PM
Triangle ABC with incenter I, incircle is tangent to BC, AC, and AB at D, E and F respectively.
DT is a diameter for the incircle, and AT meets the incircle again at point H.
Let DH and EF intersect at point J. Prove: AJ//BC.
0 replies
niwobin
Yesterday at 4:28 PM
0 replies
Weird locus problem
Sedro   1
N Yesterday at 4:20 PM by sami1618
Points $A$ and $B$ are in the coordinate plane such that $AB=2$. Let $\mathcal{H}$ denote the locus of all points $P$ in the coordinate plane satisfying $PA\cdot PB=2$, and let $M$ be the midpoint of $AB$. Points $X$ and $Y$ are on $\mathcal{H}$ such that $\angle XMY = 45^\circ$ and $MX\cdot MY=\sqrt{2}$. The value of $MX^4 + MY^4$ can be expressed in the form $\tfrac{m}{n}$, where $m$ and $n$ are relatively prime positive integers. Find $m+n$.
1 reply
Sedro
Yesterday at 3:12 AM
sami1618
Yesterday at 4:20 PM
Inequalities
sqing   4
N Yesterday at 3:35 PM by sqing
Let $ a,b,c\geq 0 , (a+8)(b+c)=9.$ Prove that
$$\frac{1}{a+1}+\frac{1}{b+1}+\frac{1}{c+1}\geq  \frac{38}{23}$$Let $ a,b,c\geq 0 , (a+2)(b+c)=3.$ Prove that
$$\frac{1}{a+1}+\frac{1}{b+1}+\frac{1}{c+1}\geq  \frac{2(2\sqrt{3}+1)}{5}$$
4 replies
sqing
Saturday at 12:50 PM
sqing
Yesterday at 3:35 PM
Find the range of 'f'
agirlhasnoname   1
N Yesterday at 2:46 PM by Mathzeus1024
Consider the triangle with vertices (1,2), (-5,-1) and (3,-2). Let Δ denote the region enclosed by the above triangle. Consider the function f:Δ-->R defined by f(x,y)= |10x - 3y|. Then the range of f is in the interval:
A)[0,36]
B)[0,47]
C)[4,47]
D)36,47]
1 reply
agirlhasnoname
May 14, 2021
Mathzeus1024
Yesterday at 2:46 PM
Function equation
hoangdinhnhatlqdqt   1
N Yesterday at 1:52 PM by Mathzeus1024
Find all functions $f:\mathbb{R}\geq 0\rightarrow \mathbb{R}\geq 0$ satisfying:
$f(f(x)-x)=2x\forall x\geq 0$
1 reply
hoangdinhnhatlqdqt
Dec 17, 2017
Mathzeus1024
Yesterday at 1:52 PM
Balkan Mathematical Olympiad 2018 P4
microsoft_office_word   32
N Apr 26, 2025 by Ilikeminecraft
Source: BMO 2018
Find all primes $p$ and $q$ such that $3p^{q-1}+1$ divides $11^p+17^p$

Proposed by Stanislav Dimitrov,Bulgaria
32 replies
microsoft_office_word
May 9, 2018
Ilikeminecraft
Apr 26, 2025
Balkan Mathematical Olympiad 2018 P4
G H J
G H BBookmark kLocked kLocked NReply
Source: BMO 2018
The post below has been deleted. Click to close.
This post has been deleted. Click here to see post.
microsoft_office_word
66 posts
#1 • 9 Y
Y by Mathuzb, samrocksnature, son7, megarnie, tiendung2006, Adventure10, Mango247, GeoKing, pomodor_ap
Find all primes $p$ and $q$ such that $3p^{q-1}+1$ divides $11^p+17^p$

Proposed by Stanislav Dimitrov,Bulgaria
This post has been edited 2 times. Last edited by microsoft_office_word, May 9, 2018, 2:29 PM
Z K Y
The post below has been deleted. Click to close.
This post has been deleted. Click here to see post.
Lamp909
98 posts
#4 • 8 Y
Y by TwoTimes3TimesSeven, Mathuzb, samrocksnature, microsoft_office_word, son7, megarnie, Adventure10, Mango247
The problem was proposed by Stanislav Dimitrov from Bulgaria
Z K Y
The post below has been deleted. Click to close.
This post has been deleted. Click here to see post.
Hamel
392 posts
#5 • 3 Y
Y by samrocksnature, Adventure10, Mango247
Is it plus or minus?
Z K Y
The post below has been deleted. Click to close.
This post has been deleted. Click here to see post.
Anar24
475 posts
#6 • 3 Y
Y by samrocksnature, Adventure10, Mango247
The problem is quite similar to APMO 2012 and Turkey EGMO TST 2016.Just looking at order and quite difficult caseworks
Z K Y
The post below has been deleted. Click to close.
This post has been deleted. Click here to see post.
Anar24
475 posts
#7 • 3 Y
Y by samrocksnature, Adventure10, Mango247
sorry forgot the statement.
Z K Y
The post below has been deleted. Click to close.
This post has been deleted. Click here to see post.
Hamel
392 posts
#8 • 3 Y
Y by samrocksnature, Adventure10, Mango247
I think it is plus. Method?
This post has been edited 3 times. Last edited by Hamel, May 9, 2018, 2:15 PM
Z K Y
The post below has been deleted. Click to close.
This post has been deleted. Click here to see post.
rightways
868 posts
#9 • 8 Y
Y by Hamel, Illuzion, Mathuzb, MathbugAOPS, Wizard_32, samrocksnature, Iora, Adventure10
Just take $3p^{q-1}+1=4^k 7^m t$. where $(t,14)=1$
and note that $t\equiv 1 \pmod p$.
And it is easy to show that $k\le 2$ and that $m\le 1$
then take both sides $\pmod p$ and finish small cases
Z K Y
The post below has been deleted. Click to close.
This post has been deleted. Click here to see post.
Illuzion
211 posts
#10 • 3 Y
Y by samrocksnature, Adventure10, Mango247
why $m \leq 1$ ?
Z K Y
The post below has been deleted. Click to close.
This post has been deleted. Click here to see post.
Hamel
392 posts
#11 • 4 Y
Y by Illuzion, samrocksnature, Adventure10, Mango247
$LTE$ yields that
Z K Y
The post below has been deleted. Click to close.
This post has been deleted. Click here to see post.
Mathuzb
431 posts
#12 • 3 Y
Y by samrocksnature, Adventure10, Mango247
rightways wrote:
Just take $3p^{q-1}+1=4^k 7^m t$. where $(t,14)=1$
and note that $t\equiv 1 \pmod p$.
And it is easy to show that $k\le 2$ and that $m\le 1$
then take both sides $\pmod p$ and finish small cases

Nice rightways, thanks
Z K Y
The post below has been deleted. Click to close.
This post has been deleted. Click here to see post.
Aiscrim
409 posts
#13 • 8 Y
Y by Illuzion, GGPiku, MihneaD, samrocksnature, Kugelmonster, Adventure10, Mango247, Gaunter_O_Dim_of_math
Indeed, $(p,q)=(3,3)$ is the only solution. If $p=2$ one can easily check that there are no solutions so suppose $p\ge 3$.

Suppose that $q$ is odd; this gives $v_2(3p^{q-1}+1)=2$. Take $r$ an odd prime dividing $3p^{q-1}+1$. Then $r$ divides $11^p+17^p$. As $11$ and $17$ are coprime, we get that $r$ divides $a^p+1$, where $a=11\cdot 17^{-1} (\mathrm{mod}\ r)$. Thus, $r$ divides $a^{2p}-1$ so the order of $a$ mod $r$ has to divide $2p$. If the order is $1$ or $p$), then $r$ divides both $a^p-1$ and $a^p+1$, so $r$ divides $2$, a contradiction. If the order is $2$, as $r$ divides $a^p+1$, we get that $r$ divides $a+1$ so $r$ divides $28$ and hence $r=7$. Otherwise the order is $2p$ so $2p$ divides $r-1$. Thus, if $3p^{q-1}+1=7^t4r_1^{a_1}....r_k^{a_k}$ for some odd distinct primes $r_i$ and some nonnegative integer $t\ge 0$ (which by LTE is less or equal to $2$), by the previous we have $r_i\equiv 1 (\mathrm{mod}\ p)$ so $3p^{q-1}+1\equiv 4,28, 196(\mathrm{mod}\ p)$ and thereby we get $p\in \{3,5,13\}$. A quick check gives $q=3$ the only possibility.

If $q=2$, we have $3p+1$ divides $11^p+17^p$. If $3p+1$ is not a power of two, proceeding as in the previous case we get that any odd prime divisor of $3p+1$ must be congruent with $1$ mod $2p$. As $3p+1$ is even, we get that $3p+1\ge 2(2p-1)$ which gives again $p=3$ so $q=3$, a contradiction. Thus $3p+1=2^k$ for some positive integer $k$. Clearly $k$ has to be even, $k=2\ell$ so $3p=(2^{\ell}-1)(2^{\ell}+1)$. As $(2^{\ell}-1,2^{\ell}+1)=1$ and $2^{\ell}-1<2^{\ell}+1$ we get that either $2^{\ell}-1=1,\ 2^{\ell}+1=3p$ or $2^{\ell}-1=3,\ 2^{\ell}+1=p$. We therefore infer that either $p=3$ (in which case again $q=3$ but we supposed $q=2$) or $p=5$. However, $p=5$ and $q=2$ would give $16$ dividing $11^5+17^5$, a contradiction.
This post has been edited 1 time. Last edited by Aiscrim, May 9, 2018, 7:32 PM
Reason: I should not attempt Olympiad problems after so long. This solution is still incomplete, but I have lost my patience. One still needs to check a little more cases in the case when q=2 because 7 can also pop up in 3p+1.
Z K Y
The post below has been deleted. Click to close.
This post has been deleted. Click here to see post.
atmargi
23 posts
#14 • 4 Y
Y by GGPiku, samrocksnature, Adventure10, Mango247
Let $r$ be an odd prime dividing $3p^{q-1}+1$, so consequently $r$ divides $11^p+17^p$. ( if $p$ is odd we have that $3p^{q-1}+1 \equiv 4 \pmod 8$, and since this number is clearly bigger than $4$, we know that it must have an odd prime divisor, and if $p$ is $2$, our number is odd, and so our hypothesis is obvious)
If $p$ is $2$ we verify by hand.
We have that $11^p \equiv -17^p \pmod r$, so $(\frac {11}{17})^{2p} \equiv 1 \pmod r$. Let the order of $(\frac {11}{17})$ be $x$. Thus, $x$ divides both $r-1$ and $2p$ and, since it can't be odd, it has to be either $2$ or $2p$. If it is $2$, we then have that $r$ divides $168$ and, since $r$ is odd and not $3$, it has to be $7$. If $x$ is $2p$, we have that $r \equiv 1 \pmod {2p}$. Taking these into consideration, we get that $3p^{q-1}+1=4^m7^ny$, where $(y,14)=1$ and $y\equiv 1 \pmod p$. By LTE, we have that $n$ is $2$ if $p=7$(we verify by hand), or $n\le 1$. Also, $m=1$.
Taking $\pmod p$, we get that $1 \equiv 4,28 \pmod p$, so $0 \equiv 3, 27 \pmod p$. Thus, $p=3$ and, after some casework, $q=3$. Thus, the only solution is $(3,3)$.
Edit:
Oops, forgot about $q=2$. If there exists a prime $r$ that divides $3p+1$ and $r$ is not $7$, we proceed as previously. if $3p+1$ is a power of $2$, then it's exponent has to be even.
Thus, $3p=(2^x-1)(2^x+1)$, and so $x$ is $1$ or $2$, which means that $p$ is either $3$ or $5$. We get a contradiction for both cases.
If $7$ divides $3p+1$, then $3p+1=2^k7$. $11^p$ is either $3$ or $1$ $\pmod 8$, and $17^p$ is $1$ $\pmod p$. Thus, $k\le 2$, and we verify the cases by hand.
This post has been edited 3 times. Last edited by atmargi, May 9, 2018, 6:48 PM
Reason: i'm stoopid
Z K Y
The post below has been deleted. Click to close.
This post has been deleted. Click here to see post.
Aryan-23
558 posts
#15 • 2 Y
Y by samrocksnature, Adventure10
Illuzion wrote:
why $m \leq 1$ ?

Hamel wrote:
$LTE$ yields that

Can someone please elaborate this ?? I am new to LTE ....
I mean how do we bound $\nu _{(7)} 3p^{q-1}+ 1$
This post has been edited 4 times. Last edited by Aryan-23, Oct 5, 2019, 7:51 PM
Z K Y
The post below has been deleted. Click to close.
This post has been deleted. Click here to see post.
arshiya381
158 posts
#16 • 2 Y
Y by samrocksnature, Adventure10
Aryan-23 wrote:
Illuzion wrote:
why $m \leq 1$ ?

Hamel wrote:
$LTE$ yields that

Can someone please elaborate this ?? I am new to LTE ....
I mean how do we bound $\nu _{(7)} 3p^{q-1}+ 1$

you dont!
you use LTE to show V7 of the right hand side is 1.
so the left hand side can not be devisible by 49.
Z K Y
The post below has been deleted. Click to close.
This post has been deleted. Click here to see post.
Aryan-23
558 posts
#17 • 3 Y
Y by samrocksnature, Adventure10, Mango247
Oh yeah , stupid me , thanks :)
Z K Y
The post below has been deleted. Click to close.
This post has been deleted. Click here to see post.
v_Enhance
6877 posts
#18 • 8 Y
Y by Mathematicsislovely, v4913, samrocksnature, amano_hina, tiendung2006, Mathlover_1, Sedro, endless_abyss
Solution from OTIS office hours:

The answer is $(3,3)$ only which works. We first dispense of several edge cases:
  • One can check $p=2$ has no solutions.
  • One can check $p=3$ has only the solution $(3,3)$.

Claim: Every prime dividing $11^p+17^p$ is either $2$, $7$, or $1 \pmod p$.
Proof. Consider a prime $r$ which divides $11^p+17^p$. Evidently $r \ne 17$. Apparently, $(-11/17)^p \equiv 1 \pmod r$, so either $-11/17 \equiv 1 \pmod r$ meaning $r \mid 28$, or $-11/17$ has order $p$, as needed. $\blacksquare$
Thus, we find $3p^{q-1}+1$ is the product of $2$, $7$, and $1 \pmod p$ primes. We now consider some new cases:
  • If $p \ne 7$ and $q \ne 2$, then $3p^{q-1}+1 \equiv 4 \pmod 8$, while $\nu_7(11^p+17^p) = 1$. Thus, we must have \[ 3p^{q-1} + 1 = 2^2 \cdot 7^{0\text{ or }1} \cdot 		\left( \text{$1 \bmod p$ primes} \right). \]This gives that either $1 \equiv 4 \pmod p$ or $1 \equiv 28 \pmod p$, but this gives $p = 3$.
  • If $p = 7$, the same analysis holds except that the exponent of $7$ must be exactly equal to $0$ (despite $\nu_7(11^p+17^p)=2$).
  • If $q=2$, the same argument works except that we need the additional observation that $11^p+17^p \not\equiv 0 \pmod 8$; hence \[ 3p + 1 = 2^{1\text{ or }2} \cdot 7^{0\text{ or }1} \cdot 		\left( \text{$1 \bmod p$ primes} \right). \]Hence we have one extra case $(p,q) = (13,2)$ to check, which fails.
This gives a complete proof that $(p,q) = (3,3)$ is the only solution.
Z K Y
The post below has been deleted. Click to close.
This post has been deleted. Click here to see post.
JapanMO2020
126 posts
#19 • 1 Y
Y by samrocksnature
microsoft_office_word wrote:
Find all primes $p$ and $q$ such that $3p^{q-1}+1$ divides $11^p+17^p$

Proposed by Stanislav Dimitrov,Bulgaria

For $p = 2$ there are no solutions. We begin like v_enhance, so if $q \lvert 11^p + 17^p$, then $q = 2, 7$ or $q \equiv 1 \pmod{p}$. Looking at $\pmod{32}$, there are no solutions so $2^4$ is maximum value of $\nu _ 2(3p^{q-1} + 1)$ and similarly $\nu _ 3(3p^{q-1} + 1) \leq 1$. Now, annoying casework gives $(p, q) = \boxed{(3, 3)}$ as the only solutions.
This post has been edited 1 time. Last edited by JapanMO2020, Jan 11, 2021, 7:29 PM
Z K Y
The post below has been deleted. Click to close.
This post has been deleted. Click here to see post.
oneteen11
180 posts
#20 • 1 Y
Y by samrocksnature
arshiya381 wrote:
Aryan-23 wrote:
Illuzion wrote:
why $m \leq 1$ ?

Hamel wrote:
$LTE$ yields that

Can someone please elaborate this ?? I am new to LTE ....
I mean how do we bound $\nu _{(7)} 3p^{q-1}+ 1$

you dont!
you use LTE to show V7 of the right hand side is 1.
so the left hand side can not be devisible by 49.

how do you get $\nu_7(11^p+17^p) = 1$?
Z K Y
The post below has been deleted. Click to close.
This post has been deleted. Click here to see post.
IAmTheHazard
5001 posts
#21
Y by
The answer is $(p,q)=(3,3)$ only, which clearly works. We can verify that for $p=2$ no solutions exist, and for $p=3$ we only have $q=3$. Henceforth assume $p>3$ We now begin with the following important claim:

Claim: If a prime $n$ divides $11^p+17^p$, then $n=2,7$ or $n \equiv 1 \pmod{2p}$.
Proof: Let $n$ be such a prime. Clearly $n \neq 11,17$. Then we have
$$11^p+17^p \equiv 0 \pmod{n} \implies \left(\frac{11}{17}\right)^p \equiv -1 \pmod{n} \implies \left(\frac{11}{17}\right)^{2p} \equiv 1.$$For convenience, let $a=\tfrac{11}{17}$. Then we have $\mathrm{ord}_n(a) \mid 2p$ but $\mathrm{ord}_n(a) \nmid p$, so either $\mathrm{ord}_n(a)=2$ or $\mathrm{ord}_n(a)=2p$. If the former is true, then we have $\tfrac{11}{17} \equiv -1 \pmod{n} \implies 28 \equiv 0 \pmod{n}$, which gives $n \in \{2,7\}$. If the latter is true, then since $\mathrm{ord}_n(a) \mid n-1$, we have $p \mid 2p \mid n-1 \implies n \equiv 1 \pmod{p}$, which is the desired conclusion. $\blacksquare$

Since $3p^{q-1}+1$ is a divisor of $11^p+17^p$, it follows that $3p^{q-1}+1$ is the product of $2,7$ and primes $1 \pmod{2p}$. Further, we have
$$11^p+17^p \equiv 3^p+1 \equiv 4 \pmod{8}$$as $p$ is odd, so $\nu_2(11^p+17^p)=2$. Now suppose that $q \neq 2$. In this case, $q-1$ is even, so $3p^{q-1}+1 \equiv 3+1 \equiv 0 \pmod{4}$, so we require $ \nu_2(3p^{q-1}+1)=2$. Further, by exponent lifting we have $\nu_7(11^p+17^p)=\nu_7(28)+\nu_7(p)$, so it follows that
$$3p^{q-1}+1=4\cdot 7^\epsilon\cdot P,$$where $P$ is the product of primes that are $1 \pmod{2p}$ and $\epsilon \in \{0,1\}$this is because if $p \neq 7$ we have $\nu_7(11^p+17^p)$ and if $p=7$ we have $7 \nmid 3p^{q-1}+1$ anyways. Note that $P \equiv 1 \pmod{2p}$. Now taking $\pmod{p}$, we have $3p^{q-1}+1 \equiv 4\cdot 7^\epsilon \pmod{p}$, which is either $4$ or $28$. But we also have $3p^{q-1}+1 \equiv 1 \pmod{p}$, so either $p \mid 3$ or $p \mid 27$, giving $p=3$. Since we supposed $p>3$ this case gives no additional solutions
If $q=2$, then we need $\nu_2(3p^{q-1}+1) \in \{1,2\}$. Similarly to the previous case, if $p=7$ then $7 \nmid 3p^{q-1}+1$, otherwise we have $\nu_7(3p^{q-1}+1)\leq 7$, so
$$3p^{q-1}+1=2\cdot2^{\epsilon_1}\cdot 7^{\epsilon_2}\cdot P,$$where $P$ is the product of primes $1 \pmod{2p}$ and $\epsilon_1,\epsilon_2 \in \{0,1\}$. LIke the previous case, this means that at least one of $\{2,4,14,28\}$ is $1 \pmod{p}$, but these respectively imply that $p \mid 1, p \mid 3, p \mid 13, p \mid 27$. Since $p>3$, the only possibility is therefore $p=13$. But we can manually check that $(p,q)=(13,2)$ fails, so $(3,3)$ is the only solution. $\blacksquare$
This post has been edited 1 time. Last edited by IAmTheHazard, Aug 30, 2021, 3:41 PM
Z K Y
The post below has been deleted. Click to close.
This post has been deleted. Click here to see post.
CT17
1481 posts
#22
Y by
Let $r$ be any prime dividing $3p^{q-1} + 1$. Clearly $r\neq 11,17$. Hence, we can write $a\equiv \frac{17}{11}\pmod{r}$ so that $a^p\equiv -1\pmod{r}$. It follows that $\text{ord}_r(a)\in\{2, 2p\}$, so either $r| 28$ or $r\equiv 1\pmod{2p}$. We have several cases.

Case 1: $q = 2$. We can manually verify that $p = 2$ does not work, so $p$ is odd. Moreover, clearly $2p+1$ does not divide $3p+1$, so the only prime divisors of $3p+1$ are $2$ and $7$. If $49 | 3p+1$, then $p\neq 7$ so $7 || 11^p + 17^p$ by LTE, a contradiction. Hence, $v_7(3p + 1) \le 1$. But since $p$ is odd, $v_2(3p+1)\le v_2\left(11^p + 17^p\right) = v_2(11 + 17) = 2$. Combining these results, we obtain $3p + 1 | 28$. However, the only solution to this is $p = 2$, which we have already checked.

Case 2: $p = 2$. We can manually verify that there are no solutions in this case.

In all remaining cases, $p$ and $q$ are odd and consequently $3p^{q-1} + 1\equiv 4\pmod{8}$.

Case 3: $p = 3$. We can manually verify that the only solution in this case is $q = 3$.

Case 4: $p = 7$. Then $7$ does not divide $3p^{q-1} + 1$, so $\frac{3\cdot 7^{q-1} + 1}{4}\equiv 1\pmod{7}$, a contradiction.

Case 5: $q\neq 2$ and $p\not\in \{2,3,7\}$. By LTE, $v_7\left(11^p + 17^p\right) = v_7(11+17) = 1$. It follows that either $\frac{3p^{q-1} + 1}{4}$ or $\frac{3p^{q-1} + 1}{28}$ is equivalent to $1$ mod $p$, a contradiction as $p\neq 3$.

In summary, the only solution is $(p,q) = \boxed{(3,3)}$.
Z K Y
The post below has been deleted. Click to close.
This post has been deleted. Click here to see post.
asdf334
7585 posts
#23
Y by
wait is the condition that q is prime necessary (other than, say, parity and divisibility reasons)
Z K Y
The post below has been deleted. Click to close.
This post has been deleted. Click here to see post.
cj13609517288
1916 posts
#24
Y by
Note: All of the $\left(\frac{11}{17}\right)$'s in the proof are the actual fraction, not the value of the Legendre/Jacobi symbol.

Lemma 1. Let $r$ be a prime that divides $3p^{q-1}+1$. Then $r\in\{2,7\}$ or $r\equiv 1\pmod{2p}$.
Proof. $r=17$ obviously fails, so \[r\mid\left(\frac{11}{17}\right)^p+1.\]If we let $a=\text{ord}_r\left(\frac{11}{17}\right)$, we have $a\mid 2p$. Also, we have that $a\nmid p$ unless $r=2$(which is one of the cases anyway), so $a\in\{2,2p\}$. If $a=2$, then \[r\mid \left(\frac{11}{17}\right)^2-1\rightarrow r\mid 121-289\rightarrow r\mid 168\rightarrow r\in\{2,3,7\}.\]But $r=3$ would imply $a=1$, so $r\in\{2,7\}$. Otherwise, $a=2p$, so $2p\mid r-1$, completing our proof.

Lemma 2. If $q\ne 2$, then $p\in\{2,3\}$.
Proof. Suppose that $p\ne 2$. Since $q$ is odd, $p^{q-1}$ is a square, so let $b^2=p^{q-1}$ and $b\in\mathbb{Z}^+$. Taking mod $8$, we have
\[\nu_2\left(3b^2+1\right)=2.\]Also,
\[\nu_7\left(3b^2+1\right)\le \nu_7\left(11^p+17^p\right)=1+\nu_7(p)=\begin{cases}1&p\ne7\\2&p=7\end{cases}.\]Thus $1\equiv 3b^2+1\pmod p$ has to be equivalent to at least one of $2^2$, $2^2\cdot 7$, and $2^2\cdot 7^2$ mod $p$. So $p\mid 3$, $p\mid 27$, or $p\mid 195$. Since $7\nmid 195$, that case doesn't work, so $p=3$, which is what we wanted.

Lemma 3. If $q=2$, then $\nu_2(3p+1)\le 3$ and $\nu_7(3p+1)\le 2$.
Proof. We have the following by LTE:
\[\nu_2(3p+1)\le\nu_2\left(11^p+17^p\right)\le\nu_2(11+17)+\nu_2(p)\le 3\]\[\nu_7(3p+1)\le\nu_7\left(11^p+17^p\right)\le\nu_7(11+17)+\nu_7(p)\le 2.\]
It then remains to casework using the cases from lemma 2.
Case 1, $p=2$: Then $11^2+17^2=121+289=410$, so \[3\cdot 2^{q-1}+1\in\{1,2,5,10,41,82,205,410\}.\]Then \[2^{q-1}\in\{0,3,27,68\},\]so there are no solutions in this case.

Case 2, $p=3$: Then
\[11^3+17^3=(11+17)\left(17^2-11\cdot 17+11^2\right)=28\cdot 223.\]Therefore,
\[3^q+1\in\{1,2,4,7,14,28,223,446,892,1561,3122,6244\}.\]We can see that
\[3^q\in\{0,1,3,6,13,27,222,445,891,1560,3121,6243\}\rightarrow 3^q\in\{3,27\}.\]But $q=1$ is not a prime, so only $(p,q)=(3,3)$ works here.

Case 3, $q=2$: Then \[3p+1\mid 11^p+17^p.\]By lemma 1, all prime factors of $3p+1$ are either $2$, $7$, or $1$ mod $2p$. In the third case, $4p+1$ is too big, so $2p+1\mid 3p+1$, absurd. So
\[3p+1=2^m 7^n\]for nonnegative integers $m,n$ where $m\le 3$ and $n\le 2$(by lemma 3). Therefore,
\[3p+1\in\{1,2,4,8,7,14,28,56,49,98,196,392\}.\]This implies that
\[p\in\{0,1,2,9,16,65\},\]all of which are impossible.

Combining all cases, our answer is just $(p,q)=\boxed{(3,3)}$, which obviously works. QED.
Z K Y
The post below has been deleted. Click to close.
This post has been deleted. Click here to see post.
HamstPan38825
8864 posts
#25
Y by
This is such an incredibly boring but also slightly difficult problem.

Let $p_1$ be a prime that divides $3p^{q-1} + 1$. Then as $p_1 \mid 11^p + 17^p$, either $p_1 \equiv 1 \pmod p$ or $p_1 \mid 28$.

The rest of the problem is finding constraints on $\nu_2(3p^{q-1} + 1)$. Notice that if $p, q$ are both odd primes, then $\nu_2(3p^{q-1} + 1) = 2$ precisely. On the other hand, $2$ is not a quadratic residue mod $7$, so this means that $7 \nmid 3p^{q-1} + 1$. But this means now that $4 \equiv 1 \pmod p$ as $$3p^{q-1} + 1 \equiv 1 \pmod p,$$so $p=3$, and correspondingly $q = 3$ too.

Now if $q = 2$, then note that $\nu_2(3p+1) \leq 2$ and $\nu_7(3p+1) \leq 1$, which upon a finite case check yields no solutions.

Thus $(3, 3)$ is the only solution.
Z K Y
The post below has been deleted. Click to close.
This post has been deleted. Click here to see post.
kimyager
8 posts
#27
Y by
HamstPan38825 wrote:
Notice that if $p, q$ are both odd primes, then $\nu_2(3p^{q-1} + 1) = 2$ precisely.

No. Try $p=3$, $q=7$ for example
Z K Y
The post below has been deleted. Click to close.
This post has been deleted. Click here to see post.
Shreyasharma
682 posts
#28
Y by
Very hard. What.

Assume that $p$ and $q$ are odd primes. Note that $\nu_2(3p^{q-1} + 1) = 2$ and hence due to size reasons we must have an odd prime factor $r$. Now consider a prime factor of $11^p + 7^p$, say $t$. Then we have,
\begin{align*}
\left(\frac{-11}{7}\right)^{2p} &\equiv 1 \pmod{t}
\end{align*}Then we have $\text{ord}_t(-11 \cdot 7^{-1}) \mid 2p$. Then either $-11 \equiv 7 \pmod{t}$ and hence $t \mid 28$, or we have $2p \mid t - 1 \iff t \equiv 1 \pmod{2p}$. Then all prime factors of $11^p + 7^p$ are $1$, $2$ or $7$ modulo $2p$. As a result all prime factors of $3p^{q-1} + 1$ are equal to $2$ or $7$ or are congruent to $1$ modulo $2p$. Now note that $\nu_7(3p^{q-1} + 1) \leq 1$ as if $p \neq 7$ it is easy to see that $\nu_7(11^p + 17^p) = 1$ else if $p = 7$, then $\nu_7(3 \cdot 7^{q-1} + 1) = 0$. Assume $7 \mid 3p^{q-1} + 1$. However then,
\begin{align*}
3p^{q-1} + 1 \equiv 0 \pmod{7}\\
p^{q-1} \equiv 2 \pmod{7}
\end{align*}However as $2$ is a NQR modulo $7$ we cannot have $7 \mid 3p^{q-1} + 1$. Thus $3p^{q-1} + 1 = 4 \cdot P$ where $P$ is the product of primes congruent to $1$ modulo $2p$. However then obviously we require $3p^{q-1} + 1 \equiv 4 \pmod{p}$ or $3 \equiv 0 \pmod{p}$ and hence $p = 3$. Now we then have,
\begin{align*}
3^q + 1 \mid 11^3 + 17^3
\end{align*}from which we find $q = 3$. Now if $q = 2$ we find,
\begin{align*}
3p + 1 \mid 11^p + 17^p
\end{align*}Note that from our claim the primes dividing $3p+1$ are $2$, $7$ or are $1$ modulo $2p$. Clearly $2p + 1 \nmid 3p + 1$ and we require $\nu_p(3p + 1) \leq 7$ and $\nu_p(3p + 1) = 2$. Hence $3p + 1 = 2^x7^y$ and a finite case check leads to no solutions.
This post has been edited 2 times. Last edited by Shreyasharma, Dec 29, 2023, 1:40 AM
Z K Y
The post below has been deleted. Click to close.
This post has been deleted. Click here to see post.
joshualiu315
2534 posts
#29
Y by
The answer is $\boxed{(p,q)=(3,3)}$, which clearly works.


Claim 1: Let $r$ be a prime that divides $3p^{q-1}+1$. Either $r=2$, $r=7$, or $r \equiv 1 \pmod{2p}$.

Proof: $r$ obviously cannot be $17$, and $r=2$ clearly works, so assume $r \neq 2, 17$ for the remainder of this proof. We write

\[r \mid \left(\frac{11}{17}\right)+1,\]
meaning the order of $\left(\frac{11}{17}\right)$ modulo $r$ divides $2p$; denote this order as $o$ Furthermore, $o$ does not divide $p$, so $o$ can be $2$ or $2p$. If $o=2$,

\[\left(\frac{11}{17}\right)^2 \equiv 1 \pmod{r} \implies -168 \equiv 0 \pmod{r} \implies r \in \{3,7\}.\]
Manually checking each case gives a contradiction for $r=3$, so $r=7$ is the only extra case here.

If $o=2p$, we have $2p \mid r-1$, which gives the last solution set. $\square$


Claim 2: If $p, q \neq 2$, then $p=3$.

Proof: Suppose that $p \neq 7$. Let $a$ be the positive integer such that $a^2 = p^{q-1}$. Notice that

\[3a^2+1 \equiv 4 \pmod{8}.\]
Also,

\[\nu_7(3a^2+1) \le \nu_7(11^p+17^p) = 1+ \nu_7(p) =1.\]
Some quick analysis yields

\[3a^2+1 \equiv 2^2, 2^2 \cdot 7 \pmod{p}\]\[\implies 1 \equiv 4 \pmod{p} \ \textbf{ or } \ 1 \equiv 28 \pmod{p}.\]
This implies that $p=3$. If $p=7$, we must analogously have

\[1 \equiv 196 \pmod{p},\]
a contradiction. $\square$


Now, we break this problem into cases:

If $p=2$, we have

\[3 \cdot 2^{q-1}+1 \mid 410\]\[\implies 2^{q-1} \in \{0,3,27,68\},\]
which is impossible.

If $p=3$, we have

\[3^q+1 \mid 6244\]\[\implies 3^q \in \{0,1,3,6,13,27,222,445,891,1560,3121,6243\}.\]
This makes the only solution $q=3$.

If $q=2$, we have

\[3p+1 \mid 11^p+17^p.\]
Claim 1 gives us that the only prime factors of $3p+1$ are $2$, $7$, or $2kp+1$ for some positive integer $k$. Clearly, the latter case is out of the question, so we have that

\[3p+1 = 2^m7^n.\]
Notice that

\[\nu_2(3p+1) \le \nu_2(11^p+17^p) \le 3\]\[\nu_2(3p+1) \le \nu_2(11^p+17^p) \le 2.\]
At this point, there are a finite amount of values for $3p+1$, which yield

\[p\in\{0,1,2,9,16,65\},\]
after some computation. This is a contradiction, so there are no solutions in this case either.

Adding up all the solutions from the three cases gives the desired answer.
This post has been edited 1 time. Last edited by joshualiu315, Feb 12, 2024, 8:27 PM
Z K Y
The post below has been deleted. Click to close.
This post has been deleted. Click here to see post.
ATGY
2502 posts
#30
Y by
We will deal with the cases \(q = 2\), \(p = 2\), and \(p = 7\) later, so assume none of these are true.

\textbf{Claim 1:} If \(r\) is a prime such that \(r \mid 3p^{q - 1} + 1\), then \(r \equiv 1 \mod{p}\) or \(r \mid 28\).

\[ r \mid 3p^{q - 1} + 1 \implies 11^p + 17^p \equiv 0 \mod{r} \implies \left(\frac{-11}{17}\right)^p \equiv 1 \mod{r} \]
Let \(k\) be the order of \(\frac{-11}{17} \mod r\). We have \(k \mid (p, r - 1) = 1, p\). If \(k = 1\), then

\[ r \mid \frac{-11}{17} - 1 \implies r \mid 28 \implies r = 2, 7 \]
If \((p, r - 1) = p\), we have \(p \mid r - 1 \implies r \equiv 1 \mod{p}\).

\textbf{Claim 2:} \(v_2(3p^{q - 1} + 1) = 2\)

Since \(q - 1\) is even and \(p\) is odd,

\[ p^{q - 1} \equiv 1 \mod{8} \implies 3p^{q - 1} + 1 \equiv 4 \mod{8} \]
So let

\[ 3p^{q - 1} + 1 = 2^2 \cdot 7^\alpha \cdot p_1^{\alpha_1} \cdots p_k^{\alpha_k} \]
where \(p_1 \equiv p_2 \equiv \cdots \equiv p_k \equiv 1 \mod{p}\).

\textbf{Claim 3:} \(p = 3\), \(q = 3\)

From LTE, we have

\[ v_7(11^p + 17^p) = v_7(28) + v_7(p) = 1 \]
Therefore \(\alpha = 0, 1\), and

\[ 2^2 \cdot 7^\alpha \cdot p_1^{\alpha_1} \cdots p_k^{\alpha_k} \equiv 28, 4 \equiv 1 \mod{p} \]
So \(p \mid 27, 3 \implies p = 3\).

So, we have

\[ 3^q + 1 \mid 11^3 + 17^3 = 6244 = 7 \cdot 4 \cdot 223 \]
which quickly yields \(q = 3\).

If \(p = 2\), we have

\[ 3 \cdot 2^q + 1 \mid 11^2 + 17^2 = 410 \]
which yields no solutions.

If \(q = 2\), we have

\[ 3p + 1 \mid 11^p + 17^p \]
Here, \(v_2(3p + 1) = 1\), which means \(14 \equiv 1 \mod{p}\), or \(p = 13\), and \(q = 2\), which doesn't work.

If \(p = 7\),

\[ v_7(11^p + 17^p) = 2 \]
so the power of 7 in \(3p^{q - 1} + 1\) can be \(0, 1, 2\), where \(0, 1\) yield the same cases as earlier and \(2\) yields

\[ 196 \equiv 1 \mod{p} \implies p \mid 195 \]
which is impossible as \(7 \nmid 195\).

So, our only solution is \((3, 3)\).
This post has been edited 1 time. Last edited by ATGY, Jul 24, 2024, 9:22 PM
Z K Y
The post below has been deleted. Click to close.
This post has been deleted. Click here to see post.
kotmhn
60 posts
#31
Y by
Let $r$ be a prime factor of $3p^{q-1}+1$. Then we have that either $r = 7$ or $r\equiv 1 \pmod{p}$
Now observe that if $r_1,r_2, \dots r_i$ be all the prime factors of $3p^{q-1}+1$ we have that all $r_i \equiv 1 \pmod{p}$.
But then as $3p^{q-1}+1 = r_1^{\alpha_1}r_2^{\alpha_2}\dots r_i^{\alpha_i} = (pk_1 + 1)^{\alpha_1}(pk_2)^{\alpha_2}\dots (pk_i)^{\alpha_i}$
Here if prime factors is greater than $3$ then we have that the term with $p$ in the product on RHS is even while on LHS it is odd contradiction.
Hence only $2,3,7$ can be the prime factors of $3p^{q-1}+1$, then a case bash gives $(3,3)$ as the only solution.
Z K Y
The post below has been deleted. Click to close.
This post has been deleted. Click here to see post.
L13832
268 posts
#32 • 2 Y
Y by alexanderhamilton124, Nobitasolvesproblems1979
Take a prime $r$ such that it divides $3p^{q-1}+1$ so $\text{ord}_{r}\left(\frac{11}{17}\right)=2$ or $2p$, so $r=2,7$ or $r\equiv 1 \pmod{2p}$.
Note that if $7\mid3p^{q-1}+1$ then we get $p^{q-1}\equiv 2\pmod{7}$ which is not possible.
Now, $\nu_7(11^p+17^p)=\nu_7(28)+\nu_7(p)=1$. We check for $p=2$ which gives us $3\cdot 2^{q-1}+1\mid 410$ which is not possible. For $p=3$ we have $3^q+1\mid 6244$, after case-bash we get $q=3$.
If $p\neq7$ then we have $\nu_7(11^p+17^p)=1$, if $p=7,$ then $\nu_7(3\cdot 7^{p-1}+1)=0$. So $3p^{q-1}+1\equiv 2^2\cdot 7, 2^2\pmod{p}$, this gives us $(p,q)=(3,3)$ to be a solution.
For $q=2$ we have $3p+1\mid 11^p+17^p$, since $3p+1$ cannot be written as $1\pmod{2p}$, so it is of the form $2^i7^j$, also we can write it as $3p+1\mid 11^p+17^p$ and because $\nu_2(3p+1)=1$ we have $p=13\implies q=2$, which does not work.
So our only answer is $\boxed{(p,q)=(3,3)}$.
Z K Y
The post below has been deleted. Click to close.
This post has been deleted. Click here to see post.
Krave37
46 posts
#33 • 1 Y
Y by S_14159
Only solution is (3,3)

Let $r$ be a prime dividing $3p^{q-1}+1$, then we get by taking the inverse $\frac{11^p}{17^p}$ congruent to 1, giving either $r$ being $2,7$ or 2p divides r-1, for the second case, $3p^{q-1}+1$ is congruent to $1$ modulo 2p, which is impossible for p odd prime. Using LTE on $11^p+17^p$ for $7$, we get that, the most $7$ power is 1 and $8$ can never divide $11^p+17^p$, this means that $3p^{q-1}+1 = 7.2^n$ where $n \leq 2$, check the remaining cases.
Z K Y
The post below has been deleted. Click to close.
This post has been deleted. Click here to see post.
Assassino9931
1346 posts
#34
Y by
Take any prime $r \geq 3$ which divides $3p^{q-1}+1$, so $r \mid 11^p + 17^p$ and thus $\text{ord}_{r}\left(11 \cdot 17^{-1}\right)=2$ or $2p$, giving $r=2,7$ or $r\equiv 1 \pmod{2p}$.

Suppose $p\geq 3$. Then $\nu_2(11^p + 17^p) = 2$ (by mod $8$) and $\nu_7(11^p + 17^p) = \nu_7(28) + \nu_7(p)$ (by Lifting the Exponent) which is $1$ if $p\neq 7$ and $2$ if $p=7$. So $\nu_7(3p^{q-1} + 1) \leq 1$ (for $p=7$ actually it is $0$) and $\nu_2(3p^{q-1} + 1) \leq 2$ Hence \[ 3p^{q-1} + 1 = 2^{\leq 2}7^{\leq 1} \
\cdot \ (\mbox{primes } \equiv 1 \pmod{2p}). \]Taking the latter mod $p$ yields $p = 2, 3, 13$. Since $11^2 + 17^2 = 2 \cdot 5 \cdot 41$, checking $3 \cdot 2^{q-1} + 1 \mid 5 \cdot 41$ gives no solution. For $11^3 + 17^3 = 28 \cdot (17^2 - 17 \cdot 11 + 11^2) = 2^2 \cdot 7 \cdot 223$ checking $\displaystyle \frac{3^{q}+1}{2} \mid 2\cdot 7 \cdot 223$ gives $q=3$, so $(3,3)$ is a solution. If $p=13 = 2\cdot 7 - 1$, then the right-hand side above must be divisible by $7$, but then mod $7$ yields $3\cdot (-1)^{q-1} + 1 \equiv 0$, contradiction for any positive integer $q$.

Remark. We did not use anywhere that $q$ is prime!
Z K Y
The post below has been deleted. Click to close.
This post has been deleted. Click here to see post.
megarnie
5607 posts
#35
Y by
Solved a while ago but forgot to post

The only solution is $\boxed{(p,q) = (3,3)}$, which works.

Claim: Any prime divisor $r\mid 11^p + 17^p$ satisfies $r\in \{2,7\}$ or $r\equiv 1\pmod{2p}$.
Proof: Let $r$ be a prime divisor of $11^p + 17^p$ and $r\ne 2,7$.

We have \[11^p + 17^p \equiv 0\pmod r\]
This implies that \[\left(\frac{11}{17}\right)^{p}\equiv -1\pmod r,\]so \[\left(\frac{11}{17}\right)^{2p}\equiv 1\pmod r\]
Thus $\mathrm{ord}_r \left(\frac{11}{17}\right)$ divides $2p$ but not $p$, so it must be either $2$ or $2p$.

If $\mathrm{ord}_r \left(\frac{11}{17}\right) = 2$, then we have \[r\mid 17^ 2- 11^2 = 168\]Since $r$ is odd, $\frac{11}{17} \not\equiv 1\pmod r$, so $r\ne 3$. This implies $r=7$, which we assumed to not be true.

If $\mathrm{ord}_r\left(\frac{11}{17}\right) = 2p$, then $2p\mid r-1$, $r\equiv 1\pmod{2p}$, as desired. $\square$

Claim: $p\ne 2,7$
Proof: If $p=2$, then \[3\cdot 2^{q-1} + 1 \mid 410\]It's easy to check this is not possible.

If $p=7$, then \[3\cdot 7^{q-1} + 1 \mid 11^7 + 17^7\]Since $22\nmid 11^7 + 17^7$, $q=2$ doesn't work. Now assume $q$ is odd. We have $\nu_2(3\cdot 7^{q-1} + 1) = 2$. So any prime factor of \[\frac{3\cdot 7^{q-1} + 1}{4}\]is $1\pmod{14}$, which implies $3\cdot 7^{q-1} + 1\equiv 4\pmod{14}$, absurd. $\square$

If $p=3$, then \[3^q + 1 \mid 11^3 + 17^3  =6244\]One can manually check that this is not the case since $3^q < 6244\implies q\le 7$.

From now on, assume $p\not\in \{2,3,7\}$.

Case 1: $q=2$
Then we have \[3p+1 \mid 11^p + 17^p\]Note that no prime that if $1\pmod{2p}$ divides $3p+1$, so the only prime factors of $3p+1$ are $2$ and $7$. However \[1\le \nu_2(3p+1) \le \nu_p(11^p + 17^p) = 2\]and \[\nu_7(3p+1)\le \nu_7(11^p + 17^p) = 1\]by LTE. So \[3p+1\in \{2,4,14,28\},\]all of which don't work.

Case 2: $q>2$
Then $p^{q-1}\equiv 1\pmod 4$, so $4\mid 3\cdot p^{q-1} + 1$. Thus by Claim 15.1, \[3\cdot p^{q-1} + 1\equiv \{4,28\}\pmod{2p}\]However, \[3\cdot p^{q-1} + 1\equiv p+1\pmod{2p},\]so $p=3$, contradiction.
This post has been edited 1 time. Last edited by megarnie, Jan 4, 2025, 1:45 PM
Z K Y
The post below has been deleted. Click to close.
This post has been deleted. Click here to see post.
Ilikeminecraft
631 posts
#36
Y by
I claim the answer is $(p, q) = \boxed{(3, 3)}.$ It is easy to check $p = 2,3,$ so we will assume that $p\geq5.$

First, I claim that if a prime $r$ satisfies $r\mid11^p + 17^p,$ then either $r = 2, 7,$ or $r\equiv1\pmod p.$

By taking modulo $r,$ we have that $\left(\frac{11}{17}\right)^{2p}\equiv1\pmod r\implies\operatorname{ord}_{r}\left(\frac{11}{17}\right)\mid(2p, r - 1)\mid2p.$ If the order is $2,$ we have that $11 + 17 \equiv 0\pmod r \implies r=2, 7.$ If the order is $2p,$ this works, and $r \equiv1\pmod p.$

Note that we can write $3p^{q - 1} + 1 = 4^k7^mt,$ where $t$ is a product of primes that are $1\pmod p.$ We can easily see that $k\leq1$ by taking modulo 8.

If $p\neq7, q\neq2,$ we have that $1 \equiv 4\cdot 7^m \pmod p$. We also know that $m \leq 1$ by taking $\nu_7(11^p + 17^p) = 1 + \nu_7(p) = 1.$ Thus, $p = 3.$

If $p = 7,$ then $3\cdot7^{q - 1} + 1 = 4t.$ By taking modulo $7$, there is a contradiction.

If $q = 2,$ then $3p+1=2^{1\text{ or }2}7^{0\text{ or }1}t.$ By taking modulo $p,$ we have that $p = 13$ is possible. However, this is impossible.
Z K Y
N Quick Reply
G
H
=
a